Radicals

Arun Ram
Department of Mathematics and Statistics
University of Melbourne
Parkville, VIC 3010 Australia
aram@unimelb.edu.au

Last update: 15 June 2012

Examples.

  1. Let 𝔽 be a field. A finite dimensional vector space is both noetherian and artinian. An infinite dimensional vector space V has Rad(V)=0, soc(V) and is neither noetherian or artinian.
  2. Let R=. Then every submodule of R R is generated by one element. The ring is noetherian but not artinian: p p2 . Rad() = Lmax Lmax = p prime p =0.

Radicals and socles

If mM,   ann(m) = { rR | rm=0 }.

  1. The annihilator of M is ann(M) = {rR | rM=0}.
  2. The radical of M is Rad(M) = Pmax Pmax, the intersection of the maximal proper submodules of M.
  3. The socle of M is soc(M) = Pmin Pmin, the sum of the simple submodules of M.
  4. The head of M is M/Rad(M).
  5. The socle series of M is 0 = soc0(M) soc1(M) where soc1(M) = M and soci(M) is determined by soci(M) soci-1(M) = soc( M soci(M) ).
  6. The radical series of M is 0 = Rad0(M) Rad1(M) where Radi(M) = Rad(Radi-1(M)).
  7. The socle length of M is the smallest positive integer n such that socn(M) = M and socn-1(M) M.
  8. The radical length of M is the smallest positive integer n such that Radn(M) = 0 and Radn-1(M) 0.
  9. The socle layers of M are sock(M) / sock-1(M).
  10. The radical layers of M are Radk-1(M) / Radk(M).

If M has socle length n then M has radical length n and socj(M) Radn-j(M), 0jn.

Let R be a ring.

  1. Rad(R) = Lmax Lmax, the intersection of the maximal left ideals of R.
  2. Rad(R) = Iprim Iprim, the intersection of the primitive two-sided ideals of R.
  3. Rad(R) = {xR | 1-axb is invertible for all a,bR}.
  4. Rad(R) contains all nilpotent ideals.

Proof.
  1. This is a restatement of the definition of Rad(R), since the submodules of RR are the left ideals of R.
  2. If M is a simple R-module and mM then ann(m) = {rR | rm=0} is a maximal left ideal of R because R/ann(m) M. The primitive ideal ann(M) = {rR | rM=0} = mM ann(m).
  3. Let sRad(R). Then R(1-x) = R since 1-x is not in any maximal left ideal. So t(1-x)=1 for some tR. So 1-t = -tx Rad(R). So 1-(1-t) = t has a left inverse, which must be 1-x. So 1-x is invertible in R. By (b) Rad(R) is an ideal and so 1-axb is invertible for every a,bR. So Rad(R) = {xR | 1-axb is invertible for all a,bR}.

    Assume 1-axb is invertible for all a,bR. Let Lmax be a maximal left ideal not containing x. Then 1=ax+l for some aR,   pLmax. So 1-ax Lmax. So Lmax=R which is a contradiction. So x is an element of every maximal left ideal. So {xR | 1-axb is invertible for all a,bR} Rad(R).
  4. Let N be a nilpotent ideal with Nk=0. If xN then xk Nk=0 and so xk=0. Then ( 1+x+x2 ++xk-1 ) (1-x) = 1 and so 1-x is invertible. Thus, since N is an ideal, 1-axb is invertible for every a,bR. Thus, by (c), NRad(R).

The proof of (bb) and (bc) of the following theorem uses:

(Nakayama's lemma) If M is a finitely generated R-module and Rad(R)M = M then M=0.

Proof.
Assume M0. Let m1,...,mk be a minimal generating set for M. Since Rad(R)M = M, mk = i=1k aimi, with aiRad(R). So (1-ak)mk = i=1k-1 aimi. But 1-ak has a left inverse in R. So mk = i=1k-1 ( 1-ak )-1 aimi, which contradicts the minimality. So M=0.

Let R be an artinian ring. Then

  1. (a) Rad(R) is the largest nilpotent ideal of R.
  2. (b) If M is a finitely generated R-module then
    1. (ba) M is noetherian and artinian,
    2. (bb) Rad(M) = Rad(R)M,
    3. (bc) soc(M) = {mM | Rad(R)m=0}.
  3. (c) R is noetherian.

Proof.
  1. (a) Let n be such that Rad(R)n = Rad(R)2n. If Rad(R)n0 then there is a minimal left ideal with Rad(R)nI 0 (since Rad(R)n Rad(R)n 0 ). Let xI, x0, be such that Rad(R)nx 0. By minimality, I = Rad(R)nx = Rad(R)n Rad(R)n x. So x=ax, with aRad(R). So (1-a)x=0. Since 1-a is invertible in R, x=0. But this is a contradiction. So Rad(R)n = 0. So Rad(R) is a nilpotent ideal.
  2. (ba) Let Mi = Rad(R)iM. Then, since M is finitely generated and R is artinian, there is a surjective homomorphism RR M. Thus M is artinian. So Mi/Mi+1 is artinian and Rad(R) acts by 0. So Mi/Mi+1 is a R/Rad(R)-module and thus Mi/Mi+1 is a finite direct sum of simple submodules. So, by (a), M has a composition series and is both noetherian and artinian.
  3. (bb) By Nakayama's lemma, Rad(R) ( M/Nmax ) = 0 for every maximal proper submodule NmaxM. So Rad(R)M Nmax for every Nmax. So Rad(R)M Rad(M).

    Since M/M1 is a finite direct sum of simple modules, Rad(M/M1) = 0. So Rad(M) M1 = Rad(R)M.
  4. (bc) The set N = {mM | Rad(R)m=0} is a submodule of M and Rad(R)N = 0. Since N is artinian, N is a finite direct sum of simple submodules. So soc(M) N.

    Nakayama's lemma implies that if S is a simple module, then Rad(R)S = 0. So Rad(R)soc(M) = 0. So soc(M) N.
  5. (c) follows from (ba).

Semisimplicity

Let M be an R-module. Then M has a simple submodule.

(Schur's lemma)

  1. If Rλ are Rμ are simple R-modules then HomR (Rλ,Rμ) = 0, if   Rλ Rμ, and EndR(Rλ) = 𝔻λ is a division ring.
  2. If M = λA^ ( Rλ ) mλ is a finite direct sum of simple modules then EndR(M) = λA^ Mmλ (𝔻λ), where 𝔻λ = EndR(Rλ) are division rings.

Proof.
  1. Let φ:RλRμ be a homomorphism. Then, since Rλ and Rμ are simple, kerφ is either 0 or Rλ, and imφ is either 0 or Rμ. So φ is either 0 or an isomorphism.
  2. If M = λA^ i=1 mλ Rλ,i, with Rλ,i Rλ, for 1imλ, then EndR(M) = λA^ i,j=1 mλ EndR( Rλ,i,Rλ,j ) = λA^ Mmλ (𝔻λ).

Let M be an R-module.

  1. soc(M)=M if and only if for every submodule NM there is a submodule NM with M=NN.
  2. Let N be a submodule of M. If soc(M)=M then soc(N)=N and soc(M/N)=M/N.

Proof.
(a) ⇐) If soc(M)M then M = soc(M)N. Let N be a simple submodule of N (the existence of N is nontrivial and uses Zorn's lemma, see Theorem ???). Then soc(M)+N soc(M), but this is a contradiction to the definition of soc(M).

⇒) Let N be a submodule of M and let N = PN=0P be the sum of the simple submodules P of M such that PN=0. Then NN=0 since, for a simple submodule P of M, PN=P or PN=0. Since N+N soc(M) = M,   N+N = M. So M=NN.

The following are equivalent:

  1. M is a finite direct sum of simple submodules.
  2. M is artinian and soc(M)=M.
  3. M is noetherian and soc(M) = M.
  4. M has a finite composition series and soc(M)=M.
  5. M is finitely generated and soc(M)=M.
  6. M is artinian and Rad(M)=0.

Proof.
The implications (a)⇔(b), (a)⇔(c), (a)⇔(d) follow directly from Proposition ??? and Proposition ???a.

(a)⇒(f) follows directly from the definitions.

(f)⇒(a): Let Ni be a finite (by DCC) number of maximal propoer submodules such that Rad(M) = Ni = 0. Then φ: M M/N1 M/Nk m ( m+N1 ,..., m+Nk ) has kerφ=0. So Mim(M) is finite length and soc(M)=M. So M is a direct sum of simple submodules.

(c)⇒(e) since M is noetherian implies that M is finitely generated.

(e)⇒(c): Let N be a submodule of M and let N be a complement. Then NM/N and thus, since M is finitely generated. Thus every submodule of M is finitely generated. So M is noetherian.

(Artin-Wedderburn) The following are equivalent:

  1. R is artinian and Rad(R)=0,
  2. RR is a finite direct sum of simple modules,
  3. R λA^ Mdλ (𝔻λ), where A^ is a finite index, dλ are positive integers, and 𝔻λ are division rings.

Proof.
(a)⇔(b) is a consequence of Proposition ???.

(a)⇐(c) is a consequence of the fact that the simple Mdλ (𝔻λ) module is 𝔻λdλ the vector space of column vectors of length dλ.

(a)⇒(c): The map Rop EndR( RR ) r φr where φr(x) = xr, for   xR, is a ring isomorphism. Thus, by Schur's lemma, Rop EndR( RR ) λA^ Mdλ (𝔻λ), and thus R λA^ Mdλ (𝔻λ) op λA^ Mdλ (𝔻λ).

Radicals and finiteness conditions for rings

Let R be a ring.

  1. The ring R is noetherian if RR is noetherian.
  2. The R is artinian if RR is artinian.
  3. A left ideal of R is a submodule of RR.
  4. An ideal I of R is primitive if I=ann(M) for a simple R-module M.

Simple and almost simple rings

  1. The ring R is primitive if 0 is a primitive ideal.
  2. The ring R is semiprimitive if Rad(R)=0.
  3. The ring R is simple if its only ideals are 0 and R.
  4. The ring R is prime if A,B are ideals with AB=0 then A=0 or B=0.
  5. An ideal P is prime if R/P is a prime ring.
  6. The ring R is semiprime if 0 is the only nilpotent ideal.

Let R be a ring and let Spec(R) be the set of prime ideals of R.

  1. R is semiprime if and only if 𝔭Spec(R) 𝔭=0.
  2. R is primitive if and only if R is a dense subring of End𝔻(U) for some 𝔻-vector space U.
  3. R is artinian and semiprime if and only if R is artinian and semiprimitive.
  4. R is artinian and primitive if and only if R is artinian and simple.
  5. R is artinian and primitive if and only if RMn(𝔻), for some n1, 𝔻 a division ring.

Burnside's theorem and Jacobson density

A subring R of End𝔻(U) is dense if for every α End𝔻(U) and every finitely generated VU there is an rR with ResVU(r) = ResVU(α). Define a topology on End𝔻(U) by making U(α,V) = {βEnd𝔻(U) | ResVU(β) = ResVU(α)} open for each αEnd𝔻(U) and each finitely generated VU. Then R is dense is End𝔻(U) if End𝔻(U) is the closure of R, R_ = End𝔻(U).

Example. Consider an infinite dimensional vector space U with basis u1,u2,.... Then End(U) M() = { infinite matrices with a finite number of nonzero entries in each column }. Let I = {finite rank elements of M()} = { αEnd(U) | im(α) is finite dimensional } and let R = {n1+l | n, lI}. Then R is a dense subring of End(U), =EndR(U) and REnd(U).

Let U be a simple R-module and let Im(R) be the image of R in End(U). Let 𝔻 = EndR(U), a division ring.

  1. Im(R) is a dense subring of End𝔻(U).
  2. If R is artinian then Im(R) = End𝔻(U).

We will show that if x1,...,xnU and αEnd𝔻(U) then there is an rR with rxi=αxi for 1in. The proof is by induction on n using the following lemma.

Let uU. Then ann(x1,...,xn)u = 0 u𝔻-span { x1,...,xn }.

Proof of Theorem 5.1.
(a) Assume the lemma and assume that x1,...,xnU and αEnd𝔻(U) are given. By the induction assumption, there is rR such that rxi = αxi, for   1in-1. If xn 𝔻-span { x1,...,xn } then, by the lemma, ann( x1,...,xn-1 )xn 0. Since ann(x1,...,xn-1)xn is a nonzero R-submodule of U and U is simple ann(x1,...,xn-1)xn = U. So lxn = (α-r)xn, for some   lann(x1,...,xn-1). Then (r-l) xi = xi, for   1in-1, and (r+l)xn = xn.

(b) Let R be artinian and let U be a simple module. Let I be a minimal element of {ann(x1,...,xk) | x1,...,xkU} and let x1,...,xk be the finite subset of U such that I = ann(x1,...,xk). Let uI. If ann(x1,...,xk)u 0 then ann(x1,...,xk,u) I and ann(x1,...,xk,u) I, a contradiction to the minimality of I. So ann(x1,...,xn)u = 0. So u𝔻-span {x1,...,xn}. So U is finite dimensional. Now (c) follows from (b).

Proof of Lemma 5.2.
⇐) Trivial.

⇒) Assume ann(x1,...,xn)u = 0. The proof is by induction on n.
  1. Case 1. If ann(x1,...,xn-1)xn = 0 then xn span{x1,...,xn-1} and so ann(x1,...,xn-1)u = 0. So uspan{x1,...,xn-1}.
  2. Case 2. If ann(x1,...,xn-1)xn 0 then ann(x1,...,xn-1)xn = U. Define an R-module homomorphism α: U U lxn lu, for   lann(x1,...,xn-1). If lxn = κxn then l-κ ann(x1,...,xn-1) ann(xn) = ann(x1,...,xn). So (l-κ)u = 0 and lu=κu which shows that α is well-defined. So α𝔻 = EndR(U).

    Now ann(x1,...,xn-1) ( u-αxn ) = 0 and so, by the induction hypothesis, u-αxn span{x1,...,xn-1}. So uspan{x1,...,xn}.

Radicals of algebras

Let A be an algebra over a field 𝔽. The radical of A is the intersection of the maximal left ideals of A, Rad(A) = Lmax Lmax.

Assume A satisfies the descending chain condition on left ideals. Then A is completely reducible if and only if Rad(A)=0.

A nilpotent ideal is an ideal I such that Ik=0 for some k>0. A nilpotent element is an element xA such that xk=0 for some k>0.

If t_: A is a trace on A then Rad(t_) = {aA | t_(ab)=0 for all bA}.

Rad(A) = Rad(t_), if t_ is the trace of a faithful representation of A.

Notes and References

These notes are originally from http://researchers.ms.unimelb.edu.au/~aram@unimelb/notespre2005.html the file http://researchers.ms.unimelb.edu.au/~aram@unimelb/Notespre2005/radicals12.25.03.pdf

References

[BouA] N. Bourbaki, Algebra I, Chapters 1-3, Elements of Mathematics, Springer-Verlag, Berlin, 1990.

[BouL] N. Bourbaki, Groupes et Algèbres de Lie, Chapitre IV, V, VI, Eléments de Mathématique, Hermann, Paris, 1968.

page history